Questions   
Sort: 
 #1
avatar+26387 
+1

The quadratic polynomial p(x) leaves a remainder of 1 on division by x - 1 or x - 2.  
The product of the roots of p(x) is 1.  
What is the sum of the roots of p(x)?

 

The quadratic polynomial is \( p(x)=ax^2+bx+c\)

\(\begin{array}{|lrcll|} \hline & p(x) &=& q_1(x)(x-1)+1 \quad | \quad p(x)=ax^2+bx+c \\ x=1: & ax^2+bx+c &=& q_1(x)(x-1)+1 \\ & a+b+c &=& q_1(x)\cdot 0 +1 \\ &\mathbf{ a+b+c} &=& \mathbf{1} \qquad (1) \\ \hline & p(x) &=& q_2(x)(x-2)+1 \quad | \quad p(x)=ax^2+bx+c \\ x=2: & p(x)=ax^2+bx+c &=& q_2(x)(x-2)+1 \\ & 4a+2b+c &=& q_2(x)\cdot 0+1 \\ & \mathbf{4a+2b+c} &=& 1 \qquad (2) \\ \hline \end{array}\)

 

We have roots \(r_1\) and \(r_2\)

\(\begin{array}{|rcll|} \hline p(x)=ax^2+bx+c &=& 0 \\ ax^2+bx+c &=& 0 \quad | \quad : a \\ x^2+\underbrace{\frac{b}{a}}_{=-(r_1+r_2)}x+\underbrace{\frac{c}{a}}_{=r_1r_2} &=& 0 \\ \hline r_1r_2= 1 &=& \frac{c}{a} \\ 1 &=& \frac{c}{a} \\ \mathbf{c} &=& \mathbf{a} \\ \hline -(r_1+r_2) &=& \frac{b}{a} \\ \mathbf{r_1+r_2} &=& \mathbf{-\frac{b}{a}} \\ \hline \end{array}\)

 

 

\(\begin{array}{|lrcll|} \hline (1): &\mathbf{ a+b+c} &=& \mathbf{1} \quad | \quad c= a \\ & a+b+a &=& 1 \\ & 2a+b &=& 1 \\ & \mathbf{b} &=& \mathbf{1-2a} \qquad (3) \\ \\ (2): & \mathbf{4a+2b+c} &=& \mathbf{1} \quad | \quad c= a \\ & 4a+2b+a &=& 1 \\ & 5a+2b &=& 1 \quad | \quad \mathbf{b=1-2a} \\ & 5a+2(1-2a) &=& 1 \\ & 5a+2-4a &=& 1 \\ & a+2 &=& 1 \\ & a &=& 1-2 \\ & \mathbf{ a } &=& \mathbf{-1} \\\\ (3): & \mathbf{b} &=& \mathbf{1-2a} \quad | \quad \mathbf{ a =-1} \\ & b &=& 1-2(-1) \\ & b &=& 1+2 \\ & \mathbf{ b } &=& \mathbf{3} \\\\ & \mathbf{c} &=& \mathbf{a} \quad | \quad \mathbf{ a =-1} \\ & \mathbf{c} &=& \mathbf{-1} \\ \hline \end{array}\)

 

\(\begin{array}{|rcll|} \hline \mathbf{r_1+r_2} &=& \mathbf{-\frac{b}{a}}\quad | \quad b=3,\ a=-1 \\ r_1+r_2 &=& -\frac{3}{-1} \\ \mathbf{r_1+r_2} &=& \mathbf{3} \\ \hline \end{array}\)

 

The sum of the roots of \(p(x)= -x^2+3x-1\) is 3

 

laugh

May 15, 2020
 #1
avatar+26387 
+1

If


\(\dfrac{1}{2} + \dfrac{1}{4} + \dfrac{1}{8} + \dfrac{1}{16} + \dfrac{1}{n} + \dfrac{1}{62} + \dfrac{1}{126} + \dfrac{1}{248} = \dfrac{495}{496}\)


 what is the value of \(n\)?

 

\(\begin{array}{|rcll|} \hline \frac{1}{2} + \frac{1}{4} + \frac{1}{8} + \frac{1}{16} + \frac{1}{n} + \frac{1}{62} + \frac{1}{126} + \frac{1}{248} &=& \frac{495}{496} \quad | \quad \times 2 \\ \frac{2}{2} + \frac{2}{4} + \frac{2}{8} + \frac{2}{16} + \frac{2}{n} + \frac{2}{62} + \frac{2}{126} + \frac{2}{248} &=& \frac{495}{248} \\ 1 + \frac{1}{2} + \frac{1}{4} + \frac{1}{8} + \frac{2}{n} + \frac{1}{31} + \frac{1}{63} + \frac{2}{248} &=& \frac{495}{248} \\ 1 + \frac{1}{2} + \frac{1}{4} + \frac{1}{8} + \frac{2}{n} + \frac{1}{31} + \frac{1}{63} &=& \frac{495}{248} - \frac{2}{248} \\ 1 + \frac{1}{2} + \frac{1}{4} + \frac{1}{8} + \frac{2}{n} + \frac{1}{31} + \frac{1}{63} &=& \frac{493}{248} \quad | \quad \times 8 \\ 8 + \frac{8}{2} + \frac{8}{4} + \frac{8}{8} + \frac{16}{n} + \frac{8}{31} + \frac{8}{63} &=& \frac{493}{31} \\ 8 + 4 + 2 + 1 + \frac{16}{n} + \frac{8}{31} + \frac{8}{63} &=& \frac{493}{31} \\ 15 + \frac{16}{n} + \frac{8}{63} &=& \frac{493}{31} - \frac{8}{31} \\ 15 + \frac{16}{n} + \frac{8}{63} &=& \frac{485}{31} \\ \frac{16}{n} + \frac{8}{63} &=& \frac{485}{31} -15 \\ \frac{16}{n} + \frac{8}{63} &=& \frac{485-15*31}{31} \\ \frac{16}{n} + \frac{8}{63} &=& \frac{20}{31} \\ \frac{16}{n} &=& \frac{20}{31} - \frac{8}{63} \\ \frac{16}{n} &=& \frac{20*63-8*31}{31*63} \\ \frac{16}{n} &=& \frac{1012}{1953} \\ \frac{1}{n} &=& \frac{1012}{16*1953} \\ \frac{1}{n} &=& \frac{1012}{31248} \\ \frac{1}{n} &=& \frac{253}{7812} \\ \mathbf{n} &=& \mathbf{\frac{7812}{253}} \\ \hline \end{array}\)

 

 

laugh

May 15, 2020
 #5
avatar+33653 
0
May 15, 2020
 #1
avatar+26387 
+1

In triangle ABC, E is on AB such that AE:EB = 3:2, and D is on AC such that AD:DC = 1:4.  
F is the intersection of BD and CE.  
Find EF:FC.

 

\(\begin{array}{lclcll} \text{Let $\vec{AB}=\vec{b}$ } && \text{Let $\vec{AC}=\vec{c}$ }\\\\ \text{Let $\dfrac{AE}{AB}=\lambda=\dfrac{3}{5} $ } && \text{Let $1-\lambda=\dfrac{2}{5} $ }\\ \text{Let $\dfrac{AD}{AC}=\mu=\dfrac{1}{5} $ } && \text{Let $1-\mu=\dfrac{4}{5} $ } \\ \text{Let $\dfrac{EF}{EC}=x $ } && \text{Let $\dfrac{FC}{EC}=1-x $ }\\ \text{Let $\dfrac{EF}{FC}=\frac{x}{1-x} $ }\\\\ \text{Let $\dfrac{DF}{DB}=\eta $ }\\\\ \text{Let $\vec{AE}=\lambda\vec{b}$ } && \text{Let $\vec{AD}=\mu\vec{c}$ }\\\\ \text{Let $\vec{EC}=\vec{AC}-\vec{AE}=\vec{c}-\lambda\vec{b}$ } &&\text{Let $\vec{DB}=\vec{AB}-\vec{AD}=\vec{b}-\mu\vec{c}$ } \\\\ \text{Let $\vec{EF}=x\vec{EC}=x(\vec{c}-\lambda\vec{b})$ } &&\text{Let $\vec{DF}=\eta\vec{DB}=\eta(\vec{b}-\mu\vec{c})$ } \\ \end{array} \)

 

\(\begin{array}{|rcll|} \hline \vec{AE} + \vec{EF} &=& \vec{AD} + \vec{DF} \\ \lambda\vec{b} + x(\vec{c}-\lambda\vec{b}) &=& \mu\vec{c} + \eta(\vec{b}-\mu\vec{c}) \\ \lambda\vec{b} + x\vec{c}-x\lambda\vec{b} &=& \mu\vec{c} + \eta\vec{b}-\eta\mu\vec{c} \\ \vec{b}(\underbrace{\lambda-x\lambda- \eta}_{=0} ) &=& \vec{c} (\underbrace{\mu-\eta\mu- x}_{=0}) \\ \hline \lambda-x\lambda- \eta &=& 0 \\ \eta &=& \lambda-x\lambda \\ \mathbf{ \eta } &=& \mathbf{ \lambda(1-x) } \\ \hline \mu-\eta\mu- x &=& 0 \\ x &=& \mu-\eta\mu \\ x &=& \mu-\lambda(1-x)\mu \\ x &=& \mu-\lambda\mu(1-x) \\ x &=& \mu-\lambda\mu + \lambda\mu x \\ x - \lambda\mu x &=& \mu-\lambda\mu \\ x (1- \lambda\mu) &=& \mu(1-\lambda) \\ \mathbf{x} &=& \mathbf{\dfrac{ \mu(1-\lambda)}{1- \lambda\mu} } \\ \hline \end{array}\)

 

\(\begin{array}{|rcll|} \hline \mathbf{x} &=& \mathbf{\dfrac{ \mu(1-\lambda)}{1- \lambda\mu} } \\\\ x &=& \dfrac{ \dfrac{1}{5}* \dfrac{2}{5}}{1- \dfrac{3}{5}*\dfrac{1}{5} } \\\\ x &=& \dfrac{ \dfrac{2}{25}}{1- \dfrac{3}{25} } \\\\ x &=& \dfrac{ \dfrac{2}{25}}{ \dfrac{22}{25} } \\\\ x &=& \dfrac{2}{22} \\\\ \mathbf{x} &=& \mathbf{\dfrac{1}{11}} \\\\ \mathbf{1-x} = 1-\dfrac{1}{11} &=& \mathbf{\dfrac{10}{11}} \\ \hline \end{array}\)

 

\(\begin{array}{|rcll|} \hline \dfrac{EF}{FC} &=& \dfrac{x}{1-x} \\\\ \dfrac{EF}{FC} &=&\dfrac{ \dfrac{1}{11} } { \dfrac{10}{11} } \\\\ \mathbf{\dfrac{EF}{FC}} &=& \mathbf{\dfrac{1}{10}} \\ \hline \end{array}\)

 

laugh

May 15, 2020

1 Online Users

avatar